Inégalité EDP

Barjovrille
Modifié (May 2022) dans Analyse
Bonjour, je bloque sur la question suivante.
Soit $\Omega \subset \mathbb{R}^N$ un domaine régulier borné. Soit $T>0$. Soit $u_0 \in L^2$.
Soit $b \in L^{\infty}(]0,T[ \times \Omega)^N$, soit $c \in L^{\infty}(]0,T[ \times \Omega)$.
On pose l'équation  dans $]0,T[ \times \Omega$
$\partial_{t} u+b \cdot \nabla u+c u-\Delta u=0$ (le point est un produit scalaire)
$u|_{\partial \Omega} =0$
$u|_{t=0}=u_0$
Montrer que si $u \in C^1([0,T],C^2(\Omega)) $  est une solution de l'équation alors on a :
$\frac{d}{d t}\|u(t)\|_{L^{2}(\Omega)}^{2}+\int_{\Omega}|\nabla u(t, x)|^{2} d x \leq\left(2\|c\|_{\infty}+\|b\|_{\infty}\right)\|u(t)\|_{L^{2}(\Omega)}^{2}$.
J'ai essayé de multiplier l'équation par $u$ et de faire des ipp en espace mais je n'arrive pas à obtenir le terme avec $||b||$ à droite.

Réponses

  • Bonjour,
    En utilisant l'inégalité $xy\leqslant \frac{x^2+y^2}2$ sur l'intégrale avec $b\cdot\nabla u$, on peut avoir $$\frac{d}{d t}\|u(t)\|_{L^{2}(\Omega)}^{2}+\int_{\Omega}|\nabla u(t, x)|^{2} d x \leq\left(2\|c\|_{\infty}+\|b\|_{\infty}^{\color{red}2}\right)\|u(t)\|_{L^{2}(\Omega)}^{2}$$ qui se rapproche de ce que tu demandes. Du coup n'y aurait-il pas un carré sur $\|b\|_{\infty}$ dans l'énoncé, et que tu aurais oublié, par hasard ? :mrgreen:
  • Bonjour @Calli.  
    Normalement on a 
    $$\frac 12 \frac{d}{d t}\|u(t)\|_{L^{2}(\Omega)}^{2}+\int_{\Omega}|\nabla u(t, x)|^{2} d x \leq\|c\|_{\infty}\|u(t)\|_{L^{2}(\Omega)}^{2}+|\int_{\Omega} (b \cdot \nabla u)u |$$
    tu fais comment pour gérer  $\int_{\Omega} (b \cdot \nabla u)u $

    pour avoir uniquement ce terme en gradient dans ta formule  ---->$ \int_{\Omega}|\nabla u(t, x)|^{2} d x$
    Le 😄 Farceur


  • Barjovrille
    Modifié (May 2022)
    Bonjour, merci pour vos réponse, je suis à l'étape que @gebrane a écrit, je ne vois pas trop comment tu as fais @Calli pour avoir ce résultat à partir de ton indication. Après il y a peut être une erreur dans l'énoncé mais alors la coquille est dans toutes les questions suivantes, mais ton résultat est quand même intéressant parce que carré ou pas carré je crois que ça n'a pas trop d'incidence sur le reste des questions. Une piste possible(dans le cas où il y a un problème dans l'énoncé) c'est que si je suppose que $div(b)$ (au sens des distribution ?)   est borné ( je me dis que ce n'est pas une hypothèse abusive je l'ai déjà rencontré sur des termes de transport de ce type) alors on à l'inégalité mais avec $div (b)$ à la place de $b$.
    Je n'ai pas encore  chercher les questions suivantes mais je vous les poste quand même si ça peut donner plus de contexte et\ou  repérer si c'est une coquille ou pas .

    question b) montrer que 
    $||u||_{L^{\infty}([0,T],L^2(\Omega))}^2 + \int_{0}^{T} ||\nabla u||_{L^2(\Omega)} dt \leq ||u_0||_{L^2(\Omega)}^2 e^{CT}$
    où $C=2||c||_{\infty}+ ||b||_{\infty}$ (lemme de Grönwall ?)

    question c) en déduire que 
    $|| \partial _t u ||_{L^2([0,T],H^{-1}(\Omega))} \leq ( ||b||_{\infty} + ||c||_{\infty} \sqrt{T+1})$   $ ||u_0||_{L^2(\Omega)}e^{CT/2}$

  • Calli
    Modifié (May 2022)
    En appliquant $xy\leqslant \frac{x^2+y^2}2$ à $(b\cdot\nabla u)u$, on a $|(b\cdot\nabla u)u|\leqslant \frac{|bu|^2+|\nabla u|^2}2$. Ainsi, $$\frac 12 \frac{d}{d t}\|u(t)\|_{L^{2}(\Omega)}^{2}+\int_{\Omega}|\nabla u(t, x)|^{2} d x \leq\|c\|_{\infty}\|u(t)\|_{L^{2}(\Omega)}^{2}+\frac 12\|b\|_{\infty}^2\|u(t)\|_{L^{2}(\Omega)}^{2}+\frac 12\int_{\Omega}|\nabla u(t, x)|^{2} d x.$$
  • Perfect @Calli
    Le 😄 Farceur


  • Ok je vois, merci !
  • Bonjour, tu as traité  comment la question b?
    Le 😄 Farceur


  • Barjovrille
    Modifié (May 2022)
    Désolé, j'ai oublié de le mettre mais il y a un carré sur la norme du nabla pour cette question b)
    on calcule
    $\frac{d}{dt}(||u(t)||_{L^2(\Omega)}^2 e^{-tC}) +||\nabla u(t)||_{L^2({\Omega})}^2 e^{-tC}$
    =$\frac{d}{dt}(||u(t)||_{L^2(\Omega)}^2) e^{-tC} - C||u(t)||_{L^2(\Omega)}^2e^{-tC} + ||\nabla u(t)||_{L^2({\Omega})}^2 e^{-tC}$
    =$(\frac{d}{dt}(||u(t)||_{L^2(\Omega)}^2)+||\nabla u(t)||_{L^2({\Omega})}^2 -C||u(t)||_{L^2(\Omega)}^2)e^{-tC} \leq 0$ ( d'après question précédente)

    on a donc
    $\frac{d}{dt}(||u(t)||_{L^2(\Omega)}^2 e^{-tC}) +||\nabla u(t)||_{L^2({\Omega})}^2 e^{-tC} \leq 0$
    en integrant en temps on obtient
    $||u(t)||_{L^2(\Omega)}^2 e^{-tC} - ||u_0||_{L^2}^2 + \int_{0}^{t} ||\nabla u(s)||_{L^2({\Omega})}^2 e^{-sC} ds \leq 0 $
    donc
    $||u(t)||_{L^2(\Omega)}^2  +\int_{0}^{t} ||\nabla u(s)||_{L^2({\Omega})}^2 e^{(t-s)C}ds \leq ||u_0||_{L^2}^2 e^{tC}$
    donc
    $||u(t)||_{L^2(\Omega)}^2  +\int_{0}^{t} ||\nabla u(s)||_{L^2({\Omega})}^2 ds \leq ||u_0||_{L^2}^2 e^{tC}$
    on passe au sup pour $t \in [0,T]$ des deux coté et on a

    $||u||_{L^{\infty}([0,T],L^2(\Omega))}^2  +\int_{0}^{T} ||\nabla u(s)||_{L^2({\Omega})}^2 ds \leq ||u_0||_{L^2}^2 e^{TC}$

    je n'ai pas encore fait la dernière question si ça t'intéresse je poste si je trouve.

  • gebrane
    Modifié (May 2022)
    Perfect.
    À vrai dire je ne sais pas (je n 'ai pas réfléchi) si le lemme de  Grönwall  peut servir la question b.
    Si tu trouves la c, tu postes.
    Le 😄 Farceur


  • Calli
    Modifié (May 2022)
    @gebrane : Le lemme de Gronwall donnerait aussi $\|u(t)\|_{L^2(\Omega)}^2  +\int_{0}^{t} \|\nabla u(s)\|_{L^2({\Omega})}^2 ds \leq \|u_0\|_{L^2}^2 e^{tC}$. En gros, @Barjovrille l'a redémontré dans ce cas particulier.

    En revanche, je ne suis pas d'accord avec le passage au sup à la fin du message de @Barjovrille. Rien ne dit que $\sup\limits_{t\in[0,T]} \|u(t)\|_{L^2(\Omega)}^2  +\sup\limits_{t\in[0,T]}\int_{0}^{t} \|\nabla u(s)\|_{L^2({\Omega})}^2 ds\leqslant \sup\limits_{t\in[0,T]} \left(\|u(t)\|_{L^2(\Omega)}^2  +\int_{0}^{t} \|\nabla u(s)\|_{L^2({\Omega})}^2 ds\right) $. Mais je ne vois pas comment conclure alors.
  • Ce n 'est pas gênant. Pour un dollar  en plus on a  $   |u||_{L^{\infty}([0,T],L^2(\Omega))}^2  +\int_{0}^{T} ||\nabla u(s)||_{L^2({\Omega})}^2 ds \leq 2||u_0||_{L^2}^2 e^{TC}$
    Le 😄 Farceur


  • Ah oui, tu as raison.
  • Merci à vous 2 pour ces précisions.
  • On attend la c  :expressionless:
    Le 😄 Farceur


  • Calli
    Modifié (May 2022)
    Ah bon ? Maintenant c'est le questionneur qui doit répondre aux questions de celui qui était venu pour répondre.  :D
  • gebrane
    Modifié (May 2022)
    @calli  Il semble qu' tu ne connais pas @Barjovrille
    Le 😄 Farceur


  • Barjovrille
    Modifié (May 2022)
    pour la question c) :smile:
    soit $t \in [0,T]$
    Soit $v \in H^1_0(\Omega)$ on fait taper l'équation contre $v$ et on a l'inégalité suivante : 
    $|\int_{\Omega} \partial _t u(t,x)v(x) dx | \leq \int_{\Omega} |b(t,x). \nabla u(t,x) v(x) | + |c(t,x)u(t,x)v(x)| + |\nabla u(t,x) . \nabla v(x)| dx$
    Après  Cauchy Schwarz et majoration par la norme infinie on a : 
    $\leq ||b||_{\infty} ||\nabla u(t) ||_{L^2(\Omega)} ||v||_{L^2(\Omega)} +||c||_{\infty} ||u(t)||_{L^2(\Omega)} ||v||_{L^2(\Omega)} + ||\nabla u(t) ||_{L^2(\Omega)}||\nabla v ||_{L^2(\Omega)}$
    la norme $L^2$ de $v$ et de $\nabla v$ est majoré par la norme $H^1_0$ de $v$  on a donc
    $|\int_{\Omega} \partial _t u(t,x)v(x) dx| \leq  (||b||_{\infty}||\nabla u(t) ||_{L^2(\Omega)} + ||c||_{\infty} ||u(t)||_{L^2(\Omega)} + ||\nabla u(t) ||_{L^2(\Omega)}||)||v||_{H^1_0(\Omega)}$

    on divise l'inégalité par $||v||_{H^1_0(\Omega)}$ et on passe au sup pour faire apparaître la norme d'opérateur ($H^{-1}$ dual de $H^1_0(\Omega)$)
    et on a 
    $|| \partial _t u(t) ||_{H^{-1}} \leq ||b||_{\infty}||\nabla u(t) ||_{L^2(\Omega)} + ||c||_{\infty} ||u(t)||_{L^2(\Omega)} + ||\nabla u(t) ||_{L^2(\Omega)}$
    donc 
    $|| \partial _t u(t) ||_{H^{-1}}^2 \leq ((||b||_{\infty}+1)||\nabla u(t) ||_{L^2(\Omega)} + ||c||_{\infty} ||u(t)||_{L^2(\Omega)})^2 $

    $(a+b)^2 \leq 2a^2 + 2b^2$ donc 

    $|| \partial _t u(t) ||_{H^{-1}}^2 \leq  2(||b||_{\infty}+1)^2||\nabla u(t) ||_{L^2(\Omega)}^2 + 2||c||_{\infty}^2 ||u(t)||_{L^2(\Omega)}^2$

    on intègre en temps 
    $|| \partial _t u ||_{L^2[0,T],H^{-1}}^2\leq  2(||b||_{\infty}+1)^2 \int_0^{T}||\nabla u(t) ||_{L^2(\Omega)}^2 dt + 2||c||_{\infty}^2 \int_0^{T} ||u(t)||_{L^2(\Omega)}^2dt$ 
    donc 
    $|| \partial _t u ||_{L^2[0,T],H^{-1}}^2\leq  4(||b||_{\infty}+1)^2 \int_0^{T}||\nabla u(t) ||_{L^2(\Omega)}^2 dt + 4T||c||_{\infty}^2||u||^2_{L^{\infty}([0,T],L^2(\Omega)}$
    on utilise la question b) (pour majorer $||\nabla u ||$ et $||u||$ )  et on passe a la racine 
    $|| \partial _t u ||_{L^2[0,T],H^{-1}}\leq \sqrt{ 4(||b||_{\infty}+1)^2 +4T||c||_{\infty}^2} ||u_0||_{L^2}e^{CT/2}$
    ça ressemble à ce qui est demandé mais je n'ai pas la bonne constante multiplicative j'ai peut être majoré trop brutalement à une étape.

    le but de l'exercice était d'établir une estimation à priori sur les solutions de l'équation donc je ne sais pas si c'est très important d'avoir la bonne constante multiplicative ?
  • Calli
    Modifié (May 2022)
    Je ne pense pas que ce soit très grave d'avoir une constante un peu différente (dans la vraie vie ; pour un devoir noté c'est un peu plus gênant). Tu as quand même trouvé une constante très similaire. Et puis, on était déjà pas raccord avec les constantes de l'énoncé dans les questions a et b ; je ne vois pas pourquoi ça se serait arrangé dans la c.   :D

    Juste, je ne vois pas pourquoi tu mets un facteur 4 à la ligne $|| \partial _t u ||_{L^2[0,T],H^{-1}}^2\leq  4(||b||_{\infty}+1)^2 \int_0^{T}||\nabla u(t) ||_{L^2(\Omega)}^2 dt + 4T||c||_{\infty}^2||u||^2_{L^{\infty}([0,T],L^2(\Omega)}$ et pas juste 2. À part ça, je ne vois pas de façon de mieux majorer.
  • Barjovrille
    Modifié (May 2022)
    Ok  je me suis dit la même chose pour ces histoires de constantes :#
    Le 4 est une erreur j'ai modifié à la dernière minute pour tenir compte de ta remarque et celle de  @gebrane pour la question b), mais je l'ai mis sur les 2 dernières lignes donc correction pour l'avant dernière ligne c'est bien un 2 et la dernière ligne c'est un 4.
Connectez-vous ou Inscrivez-vous pour répondre.